Write an equation that represents the line.

Write An Equation That Represents The Line.

Answers

Answer 1

Answer:

y = (3/4)x + 2

Step-by-step explanation:

A line (or linear function) can be represented by the equation y = mx + b where "m" is the slope and "b" is the y-intercept.

The y-intercept is when x = 0 or when the line crosses the y-axis. In the graph, notice that the line crosses at (0, 2). Thus, the y-intercept is at y = 2 and "b" must be "2".

Next, we have to find the slope, "m". The slope is defined as a change in "y" divided by a change in "x". Lets use the two points which are marked on the graph already, (4, 5) and (0, 2). We first subtract the y-values, and then divide that by the difference between corresponding x-values:

(5 - 2)/(4 - 0) = 3/4                           The slope is 3/4, so m = 3/4

Now, plug "m" and "b" into the equation:

y = (3/4)x + 2

Answer 2

Answer:

y=(3/4)x+2

Step-by-step explanation:

So first you need to find the slope (3/4). To find that you can use the rise over run method. In this case they have already chosen two points on the line, but you could use any exact points. Count how many up from 2 to 5 (in green), which is 3 then how many across (in red) which is 4. Then put the rise (3) over the run (4) so 3/4.

Then b (2) is equal to y in the y intercept which is (0,2)

Write An Equation That Represents The Line.

Related Questions

Solve for x. Round to the nearest tenth, if necessary.

Answers

Answer:

Step-by-step explanation:

You are looking for the tangent ratio, which is opposite side over adjacent side.  Using the tangent ratio of an angle of 37°.

tan37° = x/89

.75355405 = x/89

.75355405 · 89 = x/89 · 89/1

67.06 = x

rounded to 67.1

Answer:

67.1? that's what I got lol

bao mom is picking him up from the ice rink she is at home which is located at the origin (0,0) how many total units is the ice rink away from the origin explain

Answers

The total units of length for the ice rink away from the origin is 14.42 units.

How to determine the distance between the coordinates for each points?

In Mathematics and Geometry, the distance between two (2) end points that are on a coordinate plane can be calculated by using the following mathematical equation:

Distance = √[(x₂ - x₁)² + (y₂ - y₁)²]

Where:

x and y represent the data points (coordinates) on a cartesian coordinate.

Note: Ice rink is located at point (12, 8).

By substituting the given end points into the distance formula, we have the following;

Distance = √[(12 - 0)² + (8 - 0)²]

Distance = √[(12)² + (8)²]

Distance = √[144 + 64]

Distance = √208

Distance = 14.42 units.

Read more on distance here: brainly.com/question/12470464

#SPJ1

Missing information:

The question is incomplete and the complete question is shown in the attached picture.

write two different equations with variables and draw a diagram to solve the situation. For cleaning the attic Kristin was given $18. now she has $50. how much money did she have before

Answers

Once again, we get the same answer: Kristin had $32 before receiving the $18 for cleaning the attic. we solve this by forming equation and solving it

what is equation ?

An equation is a mathematical statement that shows that two expressions are equal. It usually consists of variables, constants, and mathematical operations such as addition, subtraction, multiplication, and division. Equations are used to solve problems in various fields, including mathematics, physics, engineering

In the given question,

Let's use "x" to represent the amount of money Kristin had before receiving the $18 for cleaning the attic. We can set up the following equation:

x + 18 = 50

This equation represents the total amount of money Kristin has now, which is $50, after receiving the $18 for cleaning the attic. To solve for "x", we can subtract 18 from both sides of the equation:

x + 18 - 18 = 50 - 18

x = 32

Therefore, Kristin had $32 before receiving the $18 for cleaning the attic.

Alternatively, we can also use a diagram to represent the situation. We can draw a rectangle to represent the total amount of money Kristin has now, which is $50. We can then divide the rectangle into two parts: one part representing the $18 she received for cleaning the attic, and the other part representing the amount of money she had before. We can label this unknown amount as "x", and set up the following equation:

x + 18 = 50

This equation is the same as the one we set up earlier. To solve for "x", we can again subtract 18 from both sides of the equation:

x + 18 - 18 = 50 - 18

x = 32

Once again, we get the same answer: Kristin had $32 before receiving the $18 for cleaning the attic.

To know more about equation  , visit:

https://brainly.com/question/29657983

#SPJ1

Which expression is equivalent to 6^2/7 ?

Answers

Answer: A

Step-by-step explanation:

well i did alll of them and got A

Beth has three as much money as Dennis. Dennis has $20 more than Will. If all of them had a total of $1260, how much money does Beth have?

Answers

Let's use variables to represent the amounts of money each person has:

- Let x be the amount of money that Will has.
- Then, Dennis has $20 more than Will, so Dennis has x + $20.
- Beth has three times as much money as Dennis, so Beth has 3(x + $20).

We know that the total amount of money that all three have is $1260, so we can write an equation:

x + (x + $20) + 3(x + $20) = $1260

Simplifying and solving for x:

x + x + $20 + 3x + $60 = $1260
5x + $80 = $1260
5x = $1180
x = $236

So Will has $236, Dennis has x + $20 = $256 + $20 = $276, and Beth has 3(x + $20) = 3($256) = $768.

Therefore, Beth has $768.

Answer:

Beth has $752

Step-by-step explanation:

First we lay out what we know:

Beth: 3x+20

Will: x

Dennis: x+20

Next we write the equation:

3x+20+20+x+x=1260

Then we add any numbers that can be added to each other:

5x+40=1260

Then we solve,

We first minus 40 from both sides of the equation,

5x+40=1260

-40 -40

---------------------

5x=. 1220

Last we divide 1220 by 5,

5x=1220

÷5. ÷ 5

----------------

x=244

Now that we know that x is 244 we fill in the blanks,

Beth: 732+20

Will: 244

Dennis: 244+20

Then we just solve the equations,

Beth: 732+20=752

Will: 244

Dennis: 244+20=264

That's it,

if you want to check your work you can add them all up and if they equal 1260 it's correct.

752+244+264=1260

Your welcome, have a good day/night.

-5-4-3-2
y
5
3
+ do
Mark this and return
23
Which equation represents a circle with the same
radius as the circle shown but with a center at (-1, 1)?
(x-1)2 + (y + 1)² = 16
(x-1)² + (y + 1)² = 4
(x + 1)² + (y-1)² = 4
O(x + 1)² + (y-1)² = 16
Save and Exit
Next
Submis

Answers

The equation that represents the circle is (x + 1)² + (y - 1)² = 16

Writing the equation that represents a circle

The equation of a circle with center (h,k) and radius r is:

(x - h)² + (y - k)² = r²

To find the equation of the circle with center (-1,1) and radius 4, we need to shift the center of the circle to (-1,1).

We can do this by replacing x with (x - (-1)) = (x + 1) and y with (y - 1) in the equation of the original circle. This gives:

(x + 1)² + (y - 1)² = 16

This equation represents a circle with center (-1,1) and radius 4.

Therefore, the correct option is (O) (x + 1)² + (y - 1)² = 16.

The other options do not have the correct center or radius to describe the circle.

Read more about circle qt

https://brainly.com/question/1506955


#SPJ1

Solve For X
Solve For Y

Answers

Using the properties of a parallelogram, the values of x and y are:

x = 9; y = 22.

How to Find x and y in the Parallelogram?

Since the opposite sides are parallel and equal to each other in the parallelogram above, therefore, Opposite angles are equal (or congruent) while the consecutive angles are supplementary to each other.

Therefore, we have:

6y = 180 - 48

6y = 132

Divide both sides by 6:

6y/6 = 132/6

y = 22

(5x + 3) + 6y = 180

5x + 3 + 6(22) = 180

5x + 135 = 180

5x = 180 - 135

5x = 45

x = 9

Learn more about parallelogram on:

https://brainly.com/question/29365321

#SPJ1

Someone help i really need help with this

Answers

Answer:

35 pages per minute

Step-by-step explanation:

175/5= 35

Check

35 x 5 = 175

175/35= 5

Solution to augmented matrix? Never seen a problem like this one. How do I solve it?

Answers

The row of zeros at the bottom immediately lets us conclude "infinitely many solutions". This system is consistent and dependent. This is because we have 0x+0y+0z = 0 aka 0 = 0 which is always true for any choice of x, y, and z.

The second row of values lead to the equation 0x+0y+1z = 6, aka z = 6

The first row says: 1x+6y+0z = 1 which is the same as x+6y = 1

Let's say we isolate x

x+6y = 1

x+6y-6y = 1-6y

x = 1-6y

Then we have these three values or expressions

x = 1-6yy = any real numberz = 6

All of the infinitely many solutions are of the form (x,y,z) = (1-6y, y, 6)

A lot of textbooks will use a parameter such as t, so we could write it as (1-6t, t, 6).

The choice of the letter for the parameter does not matter. I think t is most popular because it represents time. Each (x,y,z) point could represent a particle's location at any given time.

If t = 0, then we have (1, 0, 6)If t = 1, then we have (-5, 1, 6)If t = 2, then we have (-11, 2, 6)If t = 3, then we have (-17, 3, 6)

and so on.

=============================

Conclusion:

There are infinitely many solutions of the form (x,y,z) = (1-6t, t, 6) where t is any real number.

This system is consistent and dependent.

S=4lw +2wh s=92 l=9 w=2

Answers

Answer:

h=5

Step-by-step explanation:

Substituting in 92 for s,
92=4lw+2wh
Substituting in 9 for l,
92=4(9)(w)+2wh

Substituting 2 for w,

92=4(9)(2)+2(2)(h)

Simplifying by multiplying terms

92=72+4h

Subtract 72 on both sides

20=4h

Divide by 4 on both sides

5=h

PLEASE HELP ME WITH THIS IM SO CONFUSED

Answers

The probability of hitting the shaded region is 0.755 or 75.5%.

What is the probability of hitting the shaded region?

To find the probability of hitting the shaded region, we need to find the area of the shaded region and divide it by the area of the entire square.

Let's label the side length of the small square as x.

Then, we know that the diagonal of the small square is 7, so we can use the Pythagorean theorem to solve for x:

x^2 + x^2 = 7^2

2x^2 = 49

x^2 = 24.5

So the area of each white square is x^2 = 24.5 square units.

Let's label the side length of the large square as y.

Then, we know that the diagonal of the large square is 20, so we can use the Pythagorean theorem to solve for y:

y^2 + y^2 = 20^2

2y^2 = 400

y^2 = 200

So the area of the large square is y^2 = 200 square units.

Now, we can find the area of the shaded region by subtracting the area of the two white squares from the area of the large square:

Area of shaded region = Area of large square - 2 * Area of white square

= 200 - 2(24.5)

= 151

Therefore, the probability of hitting the shaded region is:

Probability = Area of shaded region / Area of entire square

Probability = 151 / 200

Probability = 0.755 or 75.5% (rounded to one decimal place)

Learn more about probability at: https://brainly.com/question/13604758

#SPJ1

Malaika has a number of candies. She can give out 12 to each of her friends and have 3 left over. or she can give 9 out to each of her friends and have 12 left over. How many friends can receive candy?

Answers

If she can give out 12 to each of her friends and have 3 left over. or she can give 9 out to each of her friends and have 12 left over, Malaika has 3 friends who can receive candies.

Let's suppose Malaika has "c" candies, and "f" is the number of friends she has.

According to the problem statement, we have two equations:

c = 12f + 3

c = 9f + 12

To solve for "f", we can set the two equations equal to each other:

12f + 3 = 9f + 12

Simplifying the equation, we get:

3f = 9

Dividing both sides by 3, we get:

f = 3

We can verify our solution by plugging "f=3" back into one of the original equations:

c = 12f + 3

c = 12(3) + 3

c = 39

So, Malaika has 39 candies in total. We can also check the other equation to make sure it is true:

c = 9f + 12

c = 9(3) + 12

c = 39

Both equations are true, so our solution of "f=3" is correct.

To learn more about equation click on,

https://brainly.com/question/941655

#SPJ1

PLS HELP ME OUT! A sporting event has a promotion in which the first 1,000 fans to enter the arena receive either a blue cap or a red cap. A random number generator is used to simulate the color of a cap given to a person where indicates a blue cap and indicates a red cap. Ten simulations, each consisting of ten random numbers, are conducted, and the results
are shown in the following table:
Based on the simulations, what is the probability that ten hats given to ten people will consist of more blue caps than red caps? a. 0.20
b. 0.40 c. 0.60 d. 0.80

Answers

The probability that ten hats given to ten people will consist of more blue caps than red caps is given as follows:

a. 0.2.

Here, we have to calculate a probability:

A probability is calculated as the division of the desired number of outcomes by the total number of outcomes in the context of a problem/experiment.

The outcomes in which there are more blue than red caps are those in which the number of zeros is greater than the number of ones, hence the number of desired outcomes is of:

2. (simulation number 7 and simulation number 10).

Hence the probability is of:

p = 2/10

p = 0.2.

More can be learned about probability at brainly.com/question/24756209

#SPJ1

Solve for x. −−9x+5<17 AND13x+25<−1

Answers

Answer: x<4/3

Step-by-step explanation: 17-5=12/9

Simplify. (x-6)^3Write your answer without using negative exponents.

Answers

Answer:

x^3-18x^2+108x-216

There is a formula for this simplification, and you can write the answer directly if you remember the formula.

Find the approximate area of the shaded region below, consisting of a right triangle with a circle cut out of it. Use 3.14 as an
approximation for pi.

O 1.254 square meters
O 312 square meters
O2, 822 square meters
O 314 square meters

Answers

Answer:

(a) 1.254 square meters

Step-by-step explanation:

We are given a figure, consisting of a right triangle with a circle cut out of it.

To Calculate : area of the shaded region below,

First Let's find the Area of the right triangle:

Area(triangle) = 1/2 × Base × Height

= 1/2 × 56 × 56

= 1/2 × 3136

= 3136/2

= 1568 m².

Now, We are diameter of the circle = 20 m

Radius of the circle = 20/2 = 10 m

Area of circle = πr²

Acc. to question we have to use 3.14 as an

approximation for pi.

Area (circle) = 3.14 × 10 × 10

Area (circle) = 3.14 × 100

Area (circle) = 314 m²

Area of shaded region = Area of triangle - Area of circle =

= 1568 m² - 314 m²

= 1.254 square meters

Therefore, The approximate area of the shaded region is 1.254 square meters.

Option (a) is the required answer

If the third and fourth term of an arithmetic sequence are -1 and -4, what are the first and second terms?

Answers

Answer: 5 and 2

Step-by-step explanation:

In an arithmetic sequence, the difference between consecutive terms is constant. Since the third and fourth terms are -1 and -4, the common difference is -4 - (-1) = -3. Therefore, the second term is -1 - (-3) = 2 and the first term is 2 - (-3) = 5. So the first and second terms of the sequence are 5 and 2 respectively.

-2x-2y=14 slecet all the ordered pairs that are solutions

Answers

The ordered pairs of the solutions to the linear expression are В. (-7, 0) C. (0, -7) and F. (3, -4)

What are the ordered pairs of the linear expression

From the question, we have the following parameters that can be used in our computation:

The linear expression -2x - 2y =14

To determine the ordered pairs of the linear expression, we set x and y to the values in the list of options and check for the true equation

Using the above as a guide, we have the following:

А. (2, 3) В. (-7, 0) C. (0, -7)

-2(2) - 2(3) = 14

-10 = 14 --- false

-2(-7) - 2(0) = 14

14 = 14 --- true

-2(0) - 2(-7) = 14

14 = 14 --- true

D. (-7, 1) E. (-1, 7) F. (3, -4)

-2(-7) - 2(1) = 14

12 = 14 --- false

-2(-1) - 2(7) = 14

-12 = 14 --- false

2(3) - 2(-4) = 14

14 = 14 --- true

Hence, the ordered pairs of the linear expression are В. (-7, 0) C. (0, -7) and F. (3, -4)

Read more about linear relation at

brainly.com/question/30318449

#SPJ1

Complete question

Select all the ordered pairs that are solutions

-2x - 2y = 14

А. (2, 3) В. (-7, 0) C. (0, -7)

D. (-7, 1) E. (-1, 7) F. (3, -4)

Bandar Industries manufactures sporting equipment. One of the company’s products is a football helmet that requires special plastic. During the quarter ending June 30, the company manufactured 3,500 helmets, using 2,485 kilograms of plastic. The plastic cost the company $16,401.



According to the standard cost card, each helmet should require 0.66 kilograms of plastic, at a cost of $7.00 per kilogram.



Required:

1. What is the standard quantity of kilograms of plastic (SQ) that is allowed to make 3,500 helmets?

2. What is the standard materials cost allowed (SQ × SP) to make 3,500 helmets?

3. What is the materials spending variance?

4. What is the materials price variance and the materials quantity variance

Answers

1. The standard quantity of plastic allowed to make 3,500 helmets is 2,310 kilograms.

2. The standard materials cost allowed to make 3,500 helmets is $16,170.

What is statistics?

Statistics is a branch of mathematics that deals with the collection, analysis, interpretation, presentation, and organization of numerical data.

1. The standard quantity of kilograms of plastic (SQ) allowed to make 3,500 helmets can be calculated as:

SQ = Standard quantity per unit × Actual output

= 0.66 kg/helmet × 3,500 helmets

= 2,310 kg

Therefore, the standard quantity of plastic allowed to make 3,500 helmets is 2,310 kilograms.

2. The standard materials cost allowed (SQ × SP) to make 3,500 helmets can be calculated as:

Standard materials cost allowed = Standard price × Standard quantity allowed

= $7.00/kg × 2,310 kg

= $16,170

Therefore, the standard materials cost allowed to make 3,500 helmets is $16,170.

3. The materials spending variance can be calculated as the difference between the actual cost incurred and the standard cost allowed:

Materials spending variance = Actual materials cost - Standard materials cost allowed

= $16,401 - $16,170

= $231 (Favorable)

Therefore, the materials spending variance is $231 (Favorable).

4. The materials price variance and the materials quantity variance can be calculated as follows:

Materials price variance = (Actual price - Standard price) × Actual quantity

= ($16,401/2,485 kg - $7.00/kg) × 2,485 kg

= $9,141.77 (Unfavorable)

Materials quantity variance = (Actual quantity - Standard quantity allowed) × Standard price

= (2,485 kg - 2,310 kg) × $7.00/kg

= $1,225 (Unfavorable)

Therefore, the materials price variance is $9,141.77 (Unfavorable) and the materials quantity variance is $1,225 (Unfavorable).

To learn more about statistics from the given link:

https://brainly.com/question/28053564

#SPJ1

Who can help me with this?

Answers

The original price of this car is equal to $6,600.

What is a percentage?

In Mathematics, a percentage can be defined as any number that is expressed as a fraction of hundred (100). This ultimately implies that, a percentage indicates the hundredth parts of any given number.

Assuming the original price is represented by the variable x, we have the following:

12.5/100 × x = x - 5,775

0.125x = x - 5,775

5,775 = (x - 0.125x)

5,775 = 0.875x

x = 5,775/0.875

x = $6,600.

In conclusion, we can logically deduce that the percentage Pat Bain paid is 87.5%.

Read more on percentage here: brainly.com/question/28009735

#SPJ1

which has the greatest rate? y = -x , y = 3/4x +1, y = 1/2x +2, y = x

Answers

The function that has the greatest rate of change is y = x

Which function has the greatest rate of change?

From the question, we have the following parameters that can be used in our computation:

y = -x ,

y = 3/4x +1,

y = 1/2x +2,

y = x

A linear function is represented as

y = mx + c

Where

rate of change = m

So, we have

y = -x , rate = -1

y = 3/4x +1, rate = 3/4

y = 1/2x +2, rate = 1/2

y = x, rate = 1

This means that the function with the highest rate is y = x

Read more about linear relation at

https://brainly.com/question/30318449

#SPJ1

Suppose the function f(t) = e^t describes the growth of a colony of bacteria, where t is hours. find the number of bacteria present at 5 hours
a) 59.598
b) 148.413
c) 8.155
d) 79. 432

Answers

The number of bacteria present at 5 hours is (b) 148.413

How can the number of bacteria present be described?

From the question, we were given,

f(t) = eᵗ

Then  f(t) = number of bacterials present at a particular time t.

f(5) = number of bacterial present at 5 hours

So, we have

f(5) = e⁵

f(5) = 148.413

Then this can be written in the whole number as 148  which implies that the second option is correct because using f(t) = eᵗ to describes the growth of a colony of bacteria will provide us with the answer

Learn more about exponential growth at:

https://brainly.com/question/13223520

#SPJ1

ACTIVITY 2: Solve the following problems completely.

Answers

1) The amount is php 340,060

2) The rate is 13%

3) The amount is  php 137,622

What is the compound interest?

Compound interest is commonly used in many types of financial products, such as savings accounts, CDs, and loans.

1)

[tex]A = P(1 + r/n)^nt\\A = 80000( 1 + 0.16)^9.75\\A = php 340,060[/tex]

2)

[tex]12300 = 9750(1 + r)^2\\12300/9750 = (1 + r)^21.26 = (1 + r)^2\\ln 1.26 = 2 ln1 + r\\ln 1.26/2 = ln1 + r\\0.12 = ln1 + r\\e^{0.12} = 1 + r\\r = e^{0.12} - 1\\r = 0.13\\r = 13%[/tex]

3)

[tex]A = 50000(1 + 0.15/2)^2 * 7\\A = php 137,622[/tex]

Learn more about compound interest:https://brainly.com/question/14295570

#SPJ1

If the median of a data set is 13 and the mean is 13, which of the following is most likely?

Select the correct answer below:

a. The data are skewed to the left.

b. The data are skewed to the right.

c. The data are symmetrical.

Answers

Answer:

C

Step-by-step explanation:

if the mean and median are the same, the graph will be symmetrical

Help 100 up for grabs Maths

Answers

Answer:

14:10

Step-by-step explanation:

We are given that the total train journey from Reading to Worle is the same for all trains

For the first train the time taken = 13:45 - 12:05

13:45
   -
12:05
--------
1 : 40
--------

So total travel time = 1 hour 40 minutes

For the third train time taken
15:05  -  13:25


15:05 = 14:05 + 0:60 since there are 60 minutes in an hour (we carry 60 from the hours part)

14:05 + 0:60 = 14:65

15:05 - 13:25 =

14:65
   -
13:25
---------
1 : 40 or 1 hour 40 minutes same as train #1

We want to find the arrival time of train #2 at Worle

Departure time of train #2 from Reading = 12:30

Time of travel = 1 hour 40 minutes

Arrival time = 12:30 + 1:40

12:30 time
   +
 1: 40  hours
----------
13:70   time
------------------
Since there are 60 minutes in 1 hour, to convert 13:70 to proper time units
subtract 60 from 70 to get the minute part of time and add 1 to 13 to get the hour part of time

13:70 = 14:10

Therefore the time that goes into the gap = 14:10

Verify

If the second train leaves Reading at 12:30 and arrives at Worle at 14:10, then time taken =

14:10 - 12:30 = 13:70 - 12:30 = 1:40 = 1 hour 40 minutes

Hope that helps and please don't hesitate to ask more questions if you have not fully understood my explanation

The answer is gonna be 14:10

Solve. Show the system of equations used and your work in solving the system.
A phone store sells Android phones for $400 and iOS phones for $650 each. This
week they sold a total of 110 phones and earned $54,000. How many Android and
how many iOS phones did they sell?

Answers

Let's assume that the number of Android phones sold is x and the number of iOS phones sold is y. Then we can write two equations based on the given information:

x + y = 110    (equation 1)

400x + 650y = 54000   (equation 2)

To solve for x and y, we can use any method of solving systems of equations. Here, we will use the substitution method:

From equation 1, we can solve for x in terms of y: x = 110 - y

Substitute x = 110 - y into equation 2 and solve for y:

400(110 - y) + 650y = 54000

44000 - 400y + 650y = 54000

250y = 10000

y = 40

Substitute y = 40 into equation 1 and solve for x:

x + 40 = 110

x = 70

Therefore, the phone store sold 70 Android phones and 40 iOS phones.

Suppose that the functions f and g are defined as follows.
Find f.g and f+ g. Then, give their domains using interval notation.

Answers

Using interval notation, the domain of f.g and f+g is [1/4, ∞).

Given, f(x)= 1/ (5x² + 3) and [tex]g(x) = \sqrt{4x-1}[/tex]

To find f.g, we substitute g(x) into f(x) and simplify:

f.g(x) = f(g(x))

[tex]=f(\sqrt{4x-1})[/tex]

[tex]=\frac{1}{5(\sqrt{4x-1})^2 +3}[/tex]

= 1/(5(4x-1)+3)

= 1 / (20x - 2)

To find f+g, we add the functions f(x) and g(x):

f+g(x) = f(x) + g(x)

= [tex]\frac{1}{5x^2+3}+\sqrt{4x-1}[/tex]

The domain of f.g and f+g is the intersection of the domains of f(x) and g(x).

The domain of f(x) is all real numbers except for the values of x that make the denominator zero, i.e., 5x² + 3 = 0. This equation has no real solutions, so the domain of f(x) is all real numbers.

The domain of g(x) is the set of all real numbers that make the argument of the square root non-negative, i.e., 4x - 1 ≥ 0. Solving this inequality, we get x ≥ 1/4.

Therefore, the domain of f.g and f+g is x ≥ 1/4.

Using interval notation, the domain of f.g and f+g is [1/4, ∞).

Learn more about domain of function here

https://brainly.com/question/13113489

#SPJ1

the price of a shirt was $25, but it is now on sale for $20. what is the percent decrease in price?

Answers

The percent decrease in price is 20%.

To calculate the percent decrease, you can use the following formula:

(Original Price - New Price) / Original Price x 100%

Plugging in the numbers, we get:

($25 - $20) / $25 x 100% = 20%

Therefore, the percent decrease in price is 20%

:)

Find the terms: a=_1=8, r=0.7

Answers

The formula of the nth term of the sequence is f(n) = 8 *0.7r^(n - 1)

Finding the terms of the sequence:

From the question, we have the following parameters that can be used in our computation:

a=_1=8, r=0.7

Express properly

So, we have

First term, a = 8

Common ratio, r = 0.7

The above definition is of a geometric sequence that has a first term of 8 and a common ratio of 0.7

Using the above as a guide, we have the following:

f(n) = a * r^(n - 1)

substitute the known values in the above equation, so, we have the following representation

f(n) = 8 *0.7r^(n - 1)

Hence, the nth term of the sequence is f(n) = 8 *0.7r^(n - 1)

Read more about sequence at

https://brainly.com/question/6561461

#SPJ1

area of traingle is what if square root 6 in height iand base is square root 24

Answers

The area of the triangle that has height of √6 and a base of √24 is calculated as: 6 square units.

How to Find the Area of a Triangle?

The area of a triangle = 1/2 * b * h, where:

h is the height of the triangle, and

b is the base length of the triangle.

Given the following:

Height of triangle = √6

Base length of the triangle = √24

Plug in the values:

Area of triangle = 1/2 * √24 * √6

= (√24 * √6) / 2

= √144 / 2

= 12/2

Area of triangle = 6 square units.

Learn more about the Area of triangle on:

https://brainly.com/question/28470545

#SPJ1

Other Questions
Use the coordinates to find the length of each sideThen find the perimeter. (Examples 1 and 2)D(1, 2), E(1, 7), F(4, 7), G(4, 2) 1. Why would President Kennedy not have taken immediate military action against the missile sites in Cuba?2. Why would the USSR want the United States to promise not to incase Cuba?3. Why would the Soviets see the removal of the US missiles in turkey as an equal exchange for removing missiles in Cuba? A model rocket starting at rest is launched straight upward. The thrust provided by the engine accelerates the rocket upward at a rate of 4 m/s/s for 15 seconds before running out of fuel. Once out of fuel, the rocket continues moving upward for awhile before falling striaght down back to earth. The engine shuts off at 450 meters high and a velocity of 60 m/s.What is the total time that the rocket is in the air?What is the maximum altitude of the rocket after the engine shuts off?The first time the rocket is 542 m above the ground will be____ after liftoff.The second time the rocket is 542 m above the ground will be___after liftoff. Find the area of a circle with a radius of 2 2start color purple, 2, end color purple. Either enter an exact answer in terms of pi or use 3. 14 3. 143, point, 14 for pi and enter your answer as a decimal 1. Do you believe the United States is becoming more secularized or morefundamentalist? Please explain your position. A rectangular patio is 10 feet by 13 feet. what is the length of the diagonal of the patio? (use pythagorean theorem: a + b = c) 6) Mary sold $192 worth of greeting cards. If she received 25% commission on her salenow much commission did she earn? Study the following examples. the verb is in italics, and the subject is underlined. as you read eachsentence, notice how the subjects and verbs agree in number. where is mary, going after the movies? (the subject is between the verb and its helper) at the top of the form are the blanks for your address. (the subject comes after the verb.) here are my letters of recommendation. (the subject comes after the verb.)in front of me are acres of grass. (the subject comes after the verb.) there was no reason for him to leave early. (the subject comes after the verb.) where is brady? (the subject comes after the verb.)there are two vacant rooms across the hall. (the subject comes after the verb.).visit learning center's owl website to complete a subject-verb agreement exercise.reflect and respond To take the action that achieves the higher or greater value refers to which ethical principle of conduct?. pls some help with this question! In how many distinct ways can the digits in the number 2,563,183,083 be arranged?(If there are any zero digits then assume they can be placed in any position) which of the following is NOT a true statement about a credit union and a traditional bank? a. they are both for-profit institutions.b. they both offer a checking accounts.c. they both pay interest on savings to their customers.d. all of the above How would this moment in the poem be different if montague had placed the entire word turning on the next line? in "the fight" The demand function for a company's product is P=26e^{-.04q} where Q is measured in thousands of units and P is measured in dollars.(a) What price should the company charge for each unit in order to sell 2500 units? (Round your answer to two decimal places.) (b) If the company prices the products at $8.50 each, how many units will sell? (Round your answer to the nearest integer.) units Sectionalism was a major causing what conflict What do you think is the significance of the fact that judaism has such an extensive list of special days that should be observed A lake near the Arctic Circle is covered by a 2-meter-thick sheet of ice during the cold winter months. When spring arrives, the warm air gradually melts the ice, causing its thickness to decrease at a constant rate. After 3 weeks , the sheet is only 1. 25 meters thick. Let y represent the ice sheet's thickness (in meters) after weeks. Which of the following information about the graph of the relationship is given? 25. 0 kg dog is trapped on a rock in the middle of a narrow river. A 66. 0-kg rescuer has assembled a swing with negligible mass that she will use to swing down and catch the trapped dog at the bottom of her swing, and then continue swinging to the other side of the river. The ledge that the rescuer swings from is 5. 0 m above the rock, which is not high enough so the rescuer and dog together can reach the other side of the river, which is 3. 0 m above the rock. However, the rescuer can use a ladder to increase the height from which she swings. What is the minimum height of the ladder the rescuer must use so both dog and rescuer make it to the other side of the river? Assume that friction and air resistance are negligible You earn $130.00 for each subscription of magazines you sell plus a salary of $90.00 per week. How many subscriptions of magazines do you need to sell in order to make at least $1000.00 each week? Pronunciation and enunciation have all of the following in common except a. Both involve how you articulate b. Both are important to successful communication e both are tied to nonverbal communication d. Both can be impacted by rate Please select the best answer from the choices provided